Question

Two firms, where TC1 (q1) = 3q12 and TC2 (q2) = 2q22. The market demand is...

Two firms, where TC1 (q1) = 3q12 and TC2 (q2) = 2q22. The market demand is p= 36-Q. Suppose the two firms collude. Determine the quantity each firm will produce and market price. What will be each firm profit?

Homework Answers

Know the answer?
Your Answer:

Post as a guest

Your Name:

What's your source?

Earn Coins

Coins can be redeemed for fabulous gifts.

Not the answer you're looking for?
Ask your own homework help question
Similar Questions
Two local manufacturing firms have a combined demand and total cost functions given by: Q =...
Two local manufacturing firms have a combined demand and total cost functions given by: Q = 105-P TC1=5Q+0.5Q12 TC2=5Q2+0.5Q22 If they cannot successfully collude and instead produce where market price equals marginal cost, what would be their total output? What would each firms profit be?
Consider two identical firms (no. 1 and no. 2) that face a linear market demand curve....
Consider two identical firms (no. 1 and no. 2) that face a linear market demand curve. Each firm has a marginal cost of zero and the two firms together face demand: P = 50 - 0.5Q, where Q = Q1 + Q2. a. Find the Cournot equilibrium Q and P for each firm. Calculate the results rounded to the second digit after the decimal point b. Find the equilibrium Q and P for each firm assuming that the firms collude...
Suppose two firms compete in selling identical widgets. They choose their output levels Q1 and Q2...
Suppose two firms compete in selling identical widgets. They choose their output levels Q1 and Q2 simultaneously and face the demand curve. P= 30 – Q, where Q = Q1 + Q2. Both firms have a marginal cost of $9. 1. Suppose that the two firms compete by simultaneously setting PRICES? What will the price be? How much will each firm produce? What will each firm’s profits be? 2. Now, continue with the price-setting assumption in (1), and assuming the...
Consider a Cournot market with two firms that have TC(Q) =5Q. Demand is given by P=...
Consider a Cournot market with two firms that have TC(Q) =5Q. Demand is given by P= 200−2(Q1+Q2). A) Find firm 1’s profit as a function of Q1 and Q2 B) Find the equilibrium price, quantity sold by each firm, and profit for each firm.
Consider a market with two identical firms. The market demand is P = 26 – 2Q,...
Consider a market with two identical firms. The market demand is P = 26 – 2Q, where Q = q1 + q2. MC1 = MC2 = 2. 1. Solve for output and price with collusion. 2. Solve for the Cournot-Nash equilibrium. 3. Now assume this market has a Stackelberg leader, Firm 1. Solve for the quantity, price, and profit for each firm. 4. Assume there is no product differentiation and the firms follow a Bertrand pricing model. Solve for the...
Suppose there are two firms operating in a market. The firms produce identical products, and the...
Suppose there are two firms operating in a market. The firms produce identical products, and the total cost for each firm is given by C = 10qi, i = 1,2, where qi is the quantity of output produced by firm i. Therefore the marginal cost for each firm is constant at MC = 10. Also, the market demand is given by P = 106 –2Q, where Q= q1 + q2 is the total industry output. The following formulas will be...
Suppose there are two firms in a market who each simultaneously choose a quantity. Firm 1’s...
Suppose there are two firms in a market who each simultaneously choose a quantity. Firm 1’s quantity is q1, and firm 2’s quantity is q2. Therefore the market quantity is Q = q1 + q2. The market demand curve is given by P = 160 - 2Q. Also, each firm has constant marginal cost equal to 10. There are no fixed costs. The marginal revenue of the two firms are given by: MR1 = 160 – 4q1 – 2q2 MR2...
Suppose that two firms compete in the same market producing homogenous products with the following inverse...
Suppose that two firms compete in the same market producing homogenous products with the following inverse demand function: P=1,000-(Q1+Q2) The cost function of each firm is given by: C1=4Q1 C2=4Q2 Suppose that the two firms engage in Bertrand price competition. What price should firm 1 set in equilibrium? What price should firm 2 set? What are the profits for each firm in equilibrium? What is the total market output? Suppose that the two firms collude in quantity, i.e., acting together...
Consider two identical firms competing in a market described by: • (Inverse) Demand: P = 50...
Consider two identical firms competing in a market described by: • (Inverse) Demand: P = 50 − Q , where Q = q1 + q2 • Cost Firm 1: C1 = 20q1 +q1^2 • Cost Firm 2: C2 = 20q2 + q2^2 a. (1 point) What is firm 1’s marginal cost? Firm 2’s marginal cost? What can you observe about these two firms? b.(2 points) What are the equilibrium price (P∗), production quantities (q∗1,q∗2), and profits(π∗1,π∗2), if these firms are...
Fill in the blanks. Consider two firms facing the demand curve: P=60-5Q where Q=Q1+Q2. The firm's...
Fill in the blanks. Consider two firms facing the demand curve: P=60-5Q where Q=Q1+Q2. The firm's cost functions are C1(Q1)=15+10Q1 and C2(Q2)=15+20Q2 Combined, the firms will produce __ units of output, of which firm 1 will produce __ units and firm 2 will produce __ units. If the firms compete, then firm 1 will produce __ units of output and firm 2 will produce __ units of output. Draw the firms' reaction curves and sho the equilibrium. Then, indicate the...